PT16.S1.Q18 (G3) - on an undeveloped street

CLambe02CLambe02 Alum Member
edited January 2016 in Logic Games 28 karma
http://7sage.com/lsat_explanations/lsat-16-section-1-game-3/
Q.18 - The correct answer choice is A - RSRT. Rule # 3 says every R has atleast 1 T adjacent to it. In order for this to be correct, the S in this answer choice should be a T. Therefore, the only correct choice I can see is E - TSRT. Can you please confirm?

Comments

  • devin.balutdevin.balut Free Trial Member
    77 karma
    Okay just trying the set up from the start.

    So we have houses 1, 3, 5, 7
    Across from houses 2, 4, 6, 8

    There's 3 types of houses. r, s, and t.
    Adjacent houses (13, 35, 57; 24, 46, 48) cannot be the same type.
    Opposite houses (12, 3,4, 56, 78) cannot be both s.
    If a house is a 'r' type than at least one house next to it will be a t (any corner r has a t next to it).

    3 = r
    6 = s

    5 cannot be a s (6 across from it is a s). 5 cannot be a r (3 next to it is a r). Therefore:

    5 = t

    ---

    Okay so last question takes away 6 = s, which therefore takes away 5 = t so we can only go through options one by one.

    a - impossible, r with no t next to it.
    b - possible, the few restrictions in place allow this.
    c - possible

    Ah I'm an idiot it's a simple misreading. You're correct in saying A is impossible. But the question is asking what can't be true, not what can be true. So A is right because it's wrong.
Sign In or Register to comment.